How to divide ${2k^3+3k^2+k-2j^3+3j^2-j}$ with $(k+1-j)$?












1














The question I had was calculating $$displaystylefrac{1}{k+1-j}sum_{i=j}^k i^2$$



Because I didn't know how to do a variable change, I did
$$frac{1}{k+1-j}sum_{i=j}^k i^2 = frac{1}{k+1-j}left(sum_{i=1}^k i^2 - sum_{i=1}^{j-1} i^2right) = frac{2k^3+3k^2+k-2j^3+3j^2-j}{6(k+1-j)}$$



The solution given used variable change and was able to cancel out $(k+1-j)$ very easily. Finally the answer was $$frac{2k^2 + 2j^2 + 2kj - j + k}{6}$$ I have verified that these two solutions are the same by multiplying the second one with $(k+1-j).$



My question is:




How do you divide an expression like ${(2k^3+3k^2+k-2j^3+3j^2-j)}$ with $(k+1-j)$? Is it even worth it in this case to get the most simplified solution?











share|cite|improve this question
























  • you can do long division: en.wikipedia.org/wiki/Long_division
    – Pink Panther
    2 days ago
















1














The question I had was calculating $$displaystylefrac{1}{k+1-j}sum_{i=j}^k i^2$$



Because I didn't know how to do a variable change, I did
$$frac{1}{k+1-j}sum_{i=j}^k i^2 = frac{1}{k+1-j}left(sum_{i=1}^k i^2 - sum_{i=1}^{j-1} i^2right) = frac{2k^3+3k^2+k-2j^3+3j^2-j}{6(k+1-j)}$$



The solution given used variable change and was able to cancel out $(k+1-j)$ very easily. Finally the answer was $$frac{2k^2 + 2j^2 + 2kj - j + k}{6}$$ I have verified that these two solutions are the same by multiplying the second one with $(k+1-j).$



My question is:




How do you divide an expression like ${(2k^3+3k^2+k-2j^3+3j^2-j)}$ with $(k+1-j)$? Is it even worth it in this case to get the most simplified solution?











share|cite|improve this question
























  • you can do long division: en.wikipedia.org/wiki/Long_division
    – Pink Panther
    2 days ago














1












1








1







The question I had was calculating $$displaystylefrac{1}{k+1-j}sum_{i=j}^k i^2$$



Because I didn't know how to do a variable change, I did
$$frac{1}{k+1-j}sum_{i=j}^k i^2 = frac{1}{k+1-j}left(sum_{i=1}^k i^2 - sum_{i=1}^{j-1} i^2right) = frac{2k^3+3k^2+k-2j^3+3j^2-j}{6(k+1-j)}$$



The solution given used variable change and was able to cancel out $(k+1-j)$ very easily. Finally the answer was $$frac{2k^2 + 2j^2 + 2kj - j + k}{6}$$ I have verified that these two solutions are the same by multiplying the second one with $(k+1-j).$



My question is:




How do you divide an expression like ${(2k^3+3k^2+k-2j^3+3j^2-j)}$ with $(k+1-j)$? Is it even worth it in this case to get the most simplified solution?











share|cite|improve this question















The question I had was calculating $$displaystylefrac{1}{k+1-j}sum_{i=j}^k i^2$$



Because I didn't know how to do a variable change, I did
$$frac{1}{k+1-j}sum_{i=j}^k i^2 = frac{1}{k+1-j}left(sum_{i=1}^k i^2 - sum_{i=1}^{j-1} i^2right) = frac{2k^3+3k^2+k-2j^3+3j^2-j}{6(k+1-j)}$$



The solution given used variable change and was able to cancel out $(k+1-j)$ very easily. Finally the answer was $$frac{2k^2 + 2j^2 + 2kj - j + k}{6}$$ I have verified that these two solutions are the same by multiplying the second one with $(k+1-j).$



My question is:




How do you divide an expression like ${(2k^3+3k^2+k-2j^3+3j^2-j)}$ with $(k+1-j)$? Is it even worth it in this case to get the most simplified solution?








algebra-precalculus






share|cite|improve this question















share|cite|improve this question













share|cite|improve this question




share|cite|improve this question








edited 2 days ago









Shubham Johri

4,414717




4,414717










asked 2 days ago









JJChaiJJChai

304




304












  • you can do long division: en.wikipedia.org/wiki/Long_division
    – Pink Panther
    2 days ago


















  • you can do long division: en.wikipedia.org/wiki/Long_division
    – Pink Panther
    2 days ago
















you can do long division: en.wikipedia.org/wiki/Long_division
– Pink Panther
2 days ago




you can do long division: en.wikipedia.org/wiki/Long_division
– Pink Panther
2 days ago










3 Answers
3






active

oldest

votes


















1














You could use synthetic division fairly easily here.



First, we consider $2k^3+3k^2+k-2j^3+3j^2-j$ as a polynomial in $k,$ and write the $4$ coefficients $$begin{array}{c|cccc} & 2 & 3 & 1 &-2j^3+3j^2-j\ & & & &\hline & & & &end{array}$$ Next, we invert the coefficients of the divisor $k+1-j$ to get $-1k+j-1,$ and write the constant term in as $$begin{array}{c|cccc} & 2 & 3 & 1 &-2j^3+3j^2-j\j-1 & & & &\hline & & & &end{array}$$ We bring down the first term to get $$begin{array}{c|cccc} & 2 & 3 & 1 &-2j^3+3j^2-j\j-1 & & & &\hline & 2 & & &end{array}$$ Now, we multiply what we've just brought down by $j-1$ and write in the result as $$begin{array}{c|cccc} & 2 & 3 & 1 &-2j^3+3j^2-j\j-1 & & 2j-2 & &\hline & 2 & & &end{array}$$ Now, adding the numbers in that column gets us $$begin{array}{c|cccc} & 2 & 3 & 1 &-2j^3+3j^2-j\j-1 & & 2j-2 & &\hline & 2 & 2j+1 & &end{array}$$ Multiply that result by $j-1$ and write it in to get $$begin{array}{c|cccc} & 2 & 3 & 1 &-2j^3+3j^2-j\j-1 & & 2j-2 & 2j^2-j-1 &\hline & 2 & 2j+1 & &end{array}$$ Adding again gives us $$begin{array}{c|cccc} & 2 & 3 & 1 &-2j^3+3j^2-j\j-1 & & 2j-2 & 2j^2-j-1 &\hline & 2 & 2j+1 & 2j^2-j &end{array}$$ Multiplying again by $j-1$ and then adding once more, we get $$begin{array}{c|cccc} & 2 & 3 & 1 &-2j^3+3j^2-j\j-1 & & 2j-2 & 2j^2-j-1 & 2j^3-3j^2+j\hline & 2 & 2j+1 & 2j^2-j &0end{array}$$ Translating back into terms of a polynomial in $k,$ this means that $$frac{2k^3+3k^2+k-2j^3+3j^2-j}{k+1-j}=2k^2+(2j+1)k+2j^2-j+frac{0}{k+1-j},$$ or more simply, $$frac{2k^3+3k^2+k-2j^3+3j^2-j}{k+1-j}=2k^2+2j^2+2jk-j+k,$$ as desired.






share|cite|improve this answer





























    2














    Try to write the polynomial in $k$ as a polynomial in $m=k+1$.



    $2k^3+3k^2+k=big[2(k+1)^3-2-6k-6k^2big]+big[3(k+1)^2-3-6kbig]+(k+1)-1\=2(k+1)^3+3(k+1)^2+(k+1)-6(1+2k+k^2)\=2m^3-3m^2+m$



    Therefore, $2m^3-3m^2+m-2j^3+3j^2-j=2(m^3-j^3)-3(m^2-j^2)+m-j\=(m-j)big[2(m^2+j^2+mj)-3(m+j)+1big]$



    Divide by $k+1-j=m-j$ and back-substitute $m$ to get,



    $$2(m^2+j^2+mj)-3(m+j)+1=2k^2+2j^2+2kj+k-j$$






    share|cite|improve this answer





























      1














      making $k-j+1=mto j=k-m+1$



      and after substitution into



      $$
      2 k^3 + 3 k^2 + k - 2 j^3 + 3 j^2 - j =m + 6 k m + 6 k^2 m - 3 m^2 - 6 k m^2 + 2 m^3
      $$






      share|cite|improve this answer























      • The OP already knows that. His question is how do you get at it
        – Shubham Johri
        2 days ago










      • @ShubhamJohri Is it OK now?
        – Cesareo
        2 days ago










      • It looks good now
        – Shubham Johri
        2 days ago











      Your Answer





      StackExchange.ifUsing("editor", function () {
      return StackExchange.using("mathjaxEditing", function () {
      StackExchange.MarkdownEditor.creationCallbacks.add(function (editor, postfix) {
      StackExchange.mathjaxEditing.prepareWmdForMathJax(editor, postfix, [["$", "$"], ["\\(","\\)"]]);
      });
      });
      }, "mathjax-editing");

      StackExchange.ready(function() {
      var channelOptions = {
      tags: "".split(" "),
      id: "69"
      };
      initTagRenderer("".split(" "), "".split(" "), channelOptions);

      StackExchange.using("externalEditor", function() {
      // Have to fire editor after snippets, if snippets enabled
      if (StackExchange.settings.snippets.snippetsEnabled) {
      StackExchange.using("snippets", function() {
      createEditor();
      });
      }
      else {
      createEditor();
      }
      });

      function createEditor() {
      StackExchange.prepareEditor({
      heartbeatType: 'answer',
      autoActivateHeartbeat: false,
      convertImagesToLinks: true,
      noModals: true,
      showLowRepImageUploadWarning: true,
      reputationToPostImages: 10,
      bindNavPrevention: true,
      postfix: "",
      imageUploader: {
      brandingHtml: "Powered by u003ca class="icon-imgur-white" href="https://imgur.com/"u003eu003c/au003e",
      contentPolicyHtml: "User contributions licensed under u003ca href="https://creativecommons.org/licenses/by-sa/3.0/"u003ecc by-sa 3.0 with attribution requiredu003c/au003e u003ca href="https://stackoverflow.com/legal/content-policy"u003e(content policy)u003c/au003e",
      allowUrls: true
      },
      noCode: true, onDemand: true,
      discardSelector: ".discard-answer"
      ,immediatelyShowMarkdownHelp:true
      });


      }
      });














      draft saved

      draft discarded


















      StackExchange.ready(
      function () {
      StackExchange.openid.initPostLogin('.new-post-login', 'https%3a%2f%2fmath.stackexchange.com%2fquestions%2f3062708%2fhow-to-divide-2k33k2k-2j33j2-j-with-k1-j%23new-answer', 'question_page');
      }
      );

      Post as a guest















      Required, but never shown

























      3 Answers
      3






      active

      oldest

      votes








      3 Answers
      3






      active

      oldest

      votes









      active

      oldest

      votes






      active

      oldest

      votes









      1














      You could use synthetic division fairly easily here.



      First, we consider $2k^3+3k^2+k-2j^3+3j^2-j$ as a polynomial in $k,$ and write the $4$ coefficients $$begin{array}{c|cccc} & 2 & 3 & 1 &-2j^3+3j^2-j\ & & & &\hline & & & &end{array}$$ Next, we invert the coefficients of the divisor $k+1-j$ to get $-1k+j-1,$ and write the constant term in as $$begin{array}{c|cccc} & 2 & 3 & 1 &-2j^3+3j^2-j\j-1 & & & &\hline & & & &end{array}$$ We bring down the first term to get $$begin{array}{c|cccc} & 2 & 3 & 1 &-2j^3+3j^2-j\j-1 & & & &\hline & 2 & & &end{array}$$ Now, we multiply what we've just brought down by $j-1$ and write in the result as $$begin{array}{c|cccc} & 2 & 3 & 1 &-2j^3+3j^2-j\j-1 & & 2j-2 & &\hline & 2 & & &end{array}$$ Now, adding the numbers in that column gets us $$begin{array}{c|cccc} & 2 & 3 & 1 &-2j^3+3j^2-j\j-1 & & 2j-2 & &\hline & 2 & 2j+1 & &end{array}$$ Multiply that result by $j-1$ and write it in to get $$begin{array}{c|cccc} & 2 & 3 & 1 &-2j^3+3j^2-j\j-1 & & 2j-2 & 2j^2-j-1 &\hline & 2 & 2j+1 & &end{array}$$ Adding again gives us $$begin{array}{c|cccc} & 2 & 3 & 1 &-2j^3+3j^2-j\j-1 & & 2j-2 & 2j^2-j-1 &\hline & 2 & 2j+1 & 2j^2-j &end{array}$$ Multiplying again by $j-1$ and then adding once more, we get $$begin{array}{c|cccc} & 2 & 3 & 1 &-2j^3+3j^2-j\j-1 & & 2j-2 & 2j^2-j-1 & 2j^3-3j^2+j\hline & 2 & 2j+1 & 2j^2-j &0end{array}$$ Translating back into terms of a polynomial in $k,$ this means that $$frac{2k^3+3k^2+k-2j^3+3j^2-j}{k+1-j}=2k^2+(2j+1)k+2j^2-j+frac{0}{k+1-j},$$ or more simply, $$frac{2k^3+3k^2+k-2j^3+3j^2-j}{k+1-j}=2k^2+2j^2+2jk-j+k,$$ as desired.






      share|cite|improve this answer


























        1














        You could use synthetic division fairly easily here.



        First, we consider $2k^3+3k^2+k-2j^3+3j^2-j$ as a polynomial in $k,$ and write the $4$ coefficients $$begin{array}{c|cccc} & 2 & 3 & 1 &-2j^3+3j^2-j\ & & & &\hline & & & &end{array}$$ Next, we invert the coefficients of the divisor $k+1-j$ to get $-1k+j-1,$ and write the constant term in as $$begin{array}{c|cccc} & 2 & 3 & 1 &-2j^3+3j^2-j\j-1 & & & &\hline & & & &end{array}$$ We bring down the first term to get $$begin{array}{c|cccc} & 2 & 3 & 1 &-2j^3+3j^2-j\j-1 & & & &\hline & 2 & & &end{array}$$ Now, we multiply what we've just brought down by $j-1$ and write in the result as $$begin{array}{c|cccc} & 2 & 3 & 1 &-2j^3+3j^2-j\j-1 & & 2j-2 & &\hline & 2 & & &end{array}$$ Now, adding the numbers in that column gets us $$begin{array}{c|cccc} & 2 & 3 & 1 &-2j^3+3j^2-j\j-1 & & 2j-2 & &\hline & 2 & 2j+1 & &end{array}$$ Multiply that result by $j-1$ and write it in to get $$begin{array}{c|cccc} & 2 & 3 & 1 &-2j^3+3j^2-j\j-1 & & 2j-2 & 2j^2-j-1 &\hline & 2 & 2j+1 & &end{array}$$ Adding again gives us $$begin{array}{c|cccc} & 2 & 3 & 1 &-2j^3+3j^2-j\j-1 & & 2j-2 & 2j^2-j-1 &\hline & 2 & 2j+1 & 2j^2-j &end{array}$$ Multiplying again by $j-1$ and then adding once more, we get $$begin{array}{c|cccc} & 2 & 3 & 1 &-2j^3+3j^2-j\j-1 & & 2j-2 & 2j^2-j-1 & 2j^3-3j^2+j\hline & 2 & 2j+1 & 2j^2-j &0end{array}$$ Translating back into terms of a polynomial in $k,$ this means that $$frac{2k^3+3k^2+k-2j^3+3j^2-j}{k+1-j}=2k^2+(2j+1)k+2j^2-j+frac{0}{k+1-j},$$ or more simply, $$frac{2k^3+3k^2+k-2j^3+3j^2-j}{k+1-j}=2k^2+2j^2+2jk-j+k,$$ as desired.






        share|cite|improve this answer
























          1












          1








          1






          You could use synthetic division fairly easily here.



          First, we consider $2k^3+3k^2+k-2j^3+3j^2-j$ as a polynomial in $k,$ and write the $4$ coefficients $$begin{array}{c|cccc} & 2 & 3 & 1 &-2j^3+3j^2-j\ & & & &\hline & & & &end{array}$$ Next, we invert the coefficients of the divisor $k+1-j$ to get $-1k+j-1,$ and write the constant term in as $$begin{array}{c|cccc} & 2 & 3 & 1 &-2j^3+3j^2-j\j-1 & & & &\hline & & & &end{array}$$ We bring down the first term to get $$begin{array}{c|cccc} & 2 & 3 & 1 &-2j^3+3j^2-j\j-1 & & & &\hline & 2 & & &end{array}$$ Now, we multiply what we've just brought down by $j-1$ and write in the result as $$begin{array}{c|cccc} & 2 & 3 & 1 &-2j^3+3j^2-j\j-1 & & 2j-2 & &\hline & 2 & & &end{array}$$ Now, adding the numbers in that column gets us $$begin{array}{c|cccc} & 2 & 3 & 1 &-2j^3+3j^2-j\j-1 & & 2j-2 & &\hline & 2 & 2j+1 & &end{array}$$ Multiply that result by $j-1$ and write it in to get $$begin{array}{c|cccc} & 2 & 3 & 1 &-2j^3+3j^2-j\j-1 & & 2j-2 & 2j^2-j-1 &\hline & 2 & 2j+1 & &end{array}$$ Adding again gives us $$begin{array}{c|cccc} & 2 & 3 & 1 &-2j^3+3j^2-j\j-1 & & 2j-2 & 2j^2-j-1 &\hline & 2 & 2j+1 & 2j^2-j &end{array}$$ Multiplying again by $j-1$ and then adding once more, we get $$begin{array}{c|cccc} & 2 & 3 & 1 &-2j^3+3j^2-j\j-1 & & 2j-2 & 2j^2-j-1 & 2j^3-3j^2+j\hline & 2 & 2j+1 & 2j^2-j &0end{array}$$ Translating back into terms of a polynomial in $k,$ this means that $$frac{2k^3+3k^2+k-2j^3+3j^2-j}{k+1-j}=2k^2+(2j+1)k+2j^2-j+frac{0}{k+1-j},$$ or more simply, $$frac{2k^3+3k^2+k-2j^3+3j^2-j}{k+1-j}=2k^2+2j^2+2jk-j+k,$$ as desired.






          share|cite|improve this answer












          You could use synthetic division fairly easily here.



          First, we consider $2k^3+3k^2+k-2j^3+3j^2-j$ as a polynomial in $k,$ and write the $4$ coefficients $$begin{array}{c|cccc} & 2 & 3 & 1 &-2j^3+3j^2-j\ & & & &\hline & & & &end{array}$$ Next, we invert the coefficients of the divisor $k+1-j$ to get $-1k+j-1,$ and write the constant term in as $$begin{array}{c|cccc} & 2 & 3 & 1 &-2j^3+3j^2-j\j-1 & & & &\hline & & & &end{array}$$ We bring down the first term to get $$begin{array}{c|cccc} & 2 & 3 & 1 &-2j^3+3j^2-j\j-1 & & & &\hline & 2 & & &end{array}$$ Now, we multiply what we've just brought down by $j-1$ and write in the result as $$begin{array}{c|cccc} & 2 & 3 & 1 &-2j^3+3j^2-j\j-1 & & 2j-2 & &\hline & 2 & & &end{array}$$ Now, adding the numbers in that column gets us $$begin{array}{c|cccc} & 2 & 3 & 1 &-2j^3+3j^2-j\j-1 & & 2j-2 & &\hline & 2 & 2j+1 & &end{array}$$ Multiply that result by $j-1$ and write it in to get $$begin{array}{c|cccc} & 2 & 3 & 1 &-2j^3+3j^2-j\j-1 & & 2j-2 & 2j^2-j-1 &\hline & 2 & 2j+1 & &end{array}$$ Adding again gives us $$begin{array}{c|cccc} & 2 & 3 & 1 &-2j^3+3j^2-j\j-1 & & 2j-2 & 2j^2-j-1 &\hline & 2 & 2j+1 & 2j^2-j &end{array}$$ Multiplying again by $j-1$ and then adding once more, we get $$begin{array}{c|cccc} & 2 & 3 & 1 &-2j^3+3j^2-j\j-1 & & 2j-2 & 2j^2-j-1 & 2j^3-3j^2+j\hline & 2 & 2j+1 & 2j^2-j &0end{array}$$ Translating back into terms of a polynomial in $k,$ this means that $$frac{2k^3+3k^2+k-2j^3+3j^2-j}{k+1-j}=2k^2+(2j+1)k+2j^2-j+frac{0}{k+1-j},$$ or more simply, $$frac{2k^3+3k^2+k-2j^3+3j^2-j}{k+1-j}=2k^2+2j^2+2jk-j+k,$$ as desired.







          share|cite|improve this answer












          share|cite|improve this answer



          share|cite|improve this answer










          answered 2 days ago









          Cameron BuieCameron Buie

          85.1k771155




          85.1k771155























              2














              Try to write the polynomial in $k$ as a polynomial in $m=k+1$.



              $2k^3+3k^2+k=big[2(k+1)^3-2-6k-6k^2big]+big[3(k+1)^2-3-6kbig]+(k+1)-1\=2(k+1)^3+3(k+1)^2+(k+1)-6(1+2k+k^2)\=2m^3-3m^2+m$



              Therefore, $2m^3-3m^2+m-2j^3+3j^2-j=2(m^3-j^3)-3(m^2-j^2)+m-j\=(m-j)big[2(m^2+j^2+mj)-3(m+j)+1big]$



              Divide by $k+1-j=m-j$ and back-substitute $m$ to get,



              $$2(m^2+j^2+mj)-3(m+j)+1=2k^2+2j^2+2kj+k-j$$






              share|cite|improve this answer


























                2














                Try to write the polynomial in $k$ as a polynomial in $m=k+1$.



                $2k^3+3k^2+k=big[2(k+1)^3-2-6k-6k^2big]+big[3(k+1)^2-3-6kbig]+(k+1)-1\=2(k+1)^3+3(k+1)^2+(k+1)-6(1+2k+k^2)\=2m^3-3m^2+m$



                Therefore, $2m^3-3m^2+m-2j^3+3j^2-j=2(m^3-j^3)-3(m^2-j^2)+m-j\=(m-j)big[2(m^2+j^2+mj)-3(m+j)+1big]$



                Divide by $k+1-j=m-j$ and back-substitute $m$ to get,



                $$2(m^2+j^2+mj)-3(m+j)+1=2k^2+2j^2+2kj+k-j$$






                share|cite|improve this answer
























                  2












                  2








                  2






                  Try to write the polynomial in $k$ as a polynomial in $m=k+1$.



                  $2k^3+3k^2+k=big[2(k+1)^3-2-6k-6k^2big]+big[3(k+1)^2-3-6kbig]+(k+1)-1\=2(k+1)^3+3(k+1)^2+(k+1)-6(1+2k+k^2)\=2m^3-3m^2+m$



                  Therefore, $2m^3-3m^2+m-2j^3+3j^2-j=2(m^3-j^3)-3(m^2-j^2)+m-j\=(m-j)big[2(m^2+j^2+mj)-3(m+j)+1big]$



                  Divide by $k+1-j=m-j$ and back-substitute $m$ to get,



                  $$2(m^2+j^2+mj)-3(m+j)+1=2k^2+2j^2+2kj+k-j$$






                  share|cite|improve this answer












                  Try to write the polynomial in $k$ as a polynomial in $m=k+1$.



                  $2k^3+3k^2+k=big[2(k+1)^3-2-6k-6k^2big]+big[3(k+1)^2-3-6kbig]+(k+1)-1\=2(k+1)^3+3(k+1)^2+(k+1)-6(1+2k+k^2)\=2m^3-3m^2+m$



                  Therefore, $2m^3-3m^2+m-2j^3+3j^2-j=2(m^3-j^3)-3(m^2-j^2)+m-j\=(m-j)big[2(m^2+j^2+mj)-3(m+j)+1big]$



                  Divide by $k+1-j=m-j$ and back-substitute $m$ to get,



                  $$2(m^2+j^2+mj)-3(m+j)+1=2k^2+2j^2+2kj+k-j$$







                  share|cite|improve this answer












                  share|cite|improve this answer



                  share|cite|improve this answer










                  answered 2 days ago









                  Shubham JohriShubham Johri

                  4,414717




                  4,414717























                      1














                      making $k-j+1=mto j=k-m+1$



                      and after substitution into



                      $$
                      2 k^3 + 3 k^2 + k - 2 j^3 + 3 j^2 - j =m + 6 k m + 6 k^2 m - 3 m^2 - 6 k m^2 + 2 m^3
                      $$






                      share|cite|improve this answer























                      • The OP already knows that. His question is how do you get at it
                        – Shubham Johri
                        2 days ago










                      • @ShubhamJohri Is it OK now?
                        – Cesareo
                        2 days ago










                      • It looks good now
                        – Shubham Johri
                        2 days ago
















                      1














                      making $k-j+1=mto j=k-m+1$



                      and after substitution into



                      $$
                      2 k^3 + 3 k^2 + k - 2 j^3 + 3 j^2 - j =m + 6 k m + 6 k^2 m - 3 m^2 - 6 k m^2 + 2 m^3
                      $$






                      share|cite|improve this answer























                      • The OP already knows that. His question is how do you get at it
                        – Shubham Johri
                        2 days ago










                      • @ShubhamJohri Is it OK now?
                        – Cesareo
                        2 days ago










                      • It looks good now
                        – Shubham Johri
                        2 days ago














                      1












                      1








                      1






                      making $k-j+1=mto j=k-m+1$



                      and after substitution into



                      $$
                      2 k^3 + 3 k^2 + k - 2 j^3 + 3 j^2 - j =m + 6 k m + 6 k^2 m - 3 m^2 - 6 k m^2 + 2 m^3
                      $$






                      share|cite|improve this answer














                      making $k-j+1=mto j=k-m+1$



                      and after substitution into



                      $$
                      2 k^3 + 3 k^2 + k - 2 j^3 + 3 j^2 - j =m + 6 k m + 6 k^2 m - 3 m^2 - 6 k m^2 + 2 m^3
                      $$







                      share|cite|improve this answer














                      share|cite|improve this answer



                      share|cite|improve this answer








                      edited 2 days ago

























                      answered 2 days ago









                      CesareoCesareo

                      8,3413516




                      8,3413516












                      • The OP already knows that. His question is how do you get at it
                        – Shubham Johri
                        2 days ago










                      • @ShubhamJohri Is it OK now?
                        – Cesareo
                        2 days ago










                      • It looks good now
                        – Shubham Johri
                        2 days ago


















                      • The OP already knows that. His question is how do you get at it
                        – Shubham Johri
                        2 days ago










                      • @ShubhamJohri Is it OK now?
                        – Cesareo
                        2 days ago










                      • It looks good now
                        – Shubham Johri
                        2 days ago
















                      The OP already knows that. His question is how do you get at it
                      – Shubham Johri
                      2 days ago




                      The OP already knows that. His question is how do you get at it
                      – Shubham Johri
                      2 days ago












                      @ShubhamJohri Is it OK now?
                      – Cesareo
                      2 days ago




                      @ShubhamJohri Is it OK now?
                      – Cesareo
                      2 days ago












                      It looks good now
                      – Shubham Johri
                      2 days ago




                      It looks good now
                      – Shubham Johri
                      2 days ago


















                      draft saved

                      draft discarded




















































                      Thanks for contributing an answer to Mathematics Stack Exchange!


                      • Please be sure to answer the question. Provide details and share your research!

                      But avoid



                      • Asking for help, clarification, or responding to other answers.

                      • Making statements based on opinion; back them up with references or personal experience.


                      Use MathJax to format equations. MathJax reference.


                      To learn more, see our tips on writing great answers.





                      Some of your past answers have not been well-received, and you're in danger of being blocked from answering.


                      Please pay close attention to the following guidance:


                      • Please be sure to answer the question. Provide details and share your research!

                      But avoid



                      • Asking for help, clarification, or responding to other answers.

                      • Making statements based on opinion; back them up with references or personal experience.


                      To learn more, see our tips on writing great answers.




                      draft saved


                      draft discarded














                      StackExchange.ready(
                      function () {
                      StackExchange.openid.initPostLogin('.new-post-login', 'https%3a%2f%2fmath.stackexchange.com%2fquestions%2f3062708%2fhow-to-divide-2k33k2k-2j33j2-j-with-k1-j%23new-answer', 'question_page');
                      }
                      );

                      Post as a guest















                      Required, but never shown





















































                      Required, but never shown














                      Required, but never shown












                      Required, but never shown







                      Required, but never shown

































                      Required, but never shown














                      Required, but never shown












                      Required, but never shown







                      Required, but never shown







                      Popular posts from this blog

                      Mario Kart Wii

                      What does “Dominus providebit” mean?

                      Antonio Litta Visconti Arese